§1. Bất đẳng thức

dbrby

cho a,b,c > 0 thỏa mãn \(ab+bc+ca=3\) . Cmr: \(\frac{a^3}{b^2+3}+\frac{b^3}{c^2+3}+\frac{c^3}{a^2+3}\ge\frac{3}{4}\)

Akai Haruma
18 tháng 8 2019 lúc 18:46

Lời giải:

Ta có:
\(\text{VT}=\frac{a^3}{b^2+3}+\frac{b^3}{c^2+3}+\frac{c^3}{a^2+3}=\frac{a^3}{b^2+ab+bc+ac}+\frac{b^3}{c^2+ab+bc+ac}+\frac{c^3}{a^2+ab+bc+ac}\)

\(=\frac{a^3}{(b+a)(b+c)}+\frac{b^3}{(c+a)(c+b)}+\frac{c^3}{(a+b)(a+c)}\)

Áp dụng BĐT AM-GM:
\(\frac{a^3}{(b+a)(b+c)}+\frac{b+a}{8}+\frac{b+c}{8}\geq 3\sqrt[3]{\frac{a^3}{8.8}}=\frac{3a}{4}\)

\(\frac{b^3}{(c+a)(c+b)}+\frac{c+a}{8}+\frac{c+b}{8}\geq \frac{3b}{4}\)

\(\frac{c^3}{(a+b)(a+c)}+\frac{a+b}{8}+\frac{a+c}{8}\geq \frac{3c}{4}\)

Cộng theo vế và rút gọn thu được:

\(\text{VT}\geq \frac{a+b+c}{4}\)

Tiếp tục áp dụng BĐT AM-GM: \((a+b+c)^2\geq 3(ab+bc+ac)=9\Rightarrow a+b+c\geq 3\)

Do đó: \(\text{VT}\geq \frac{3}{4}\) (đpcm)

Dấu "=" xảy ra khi $a=b=c=1$

Bình luận (0)

Các câu hỏi tương tự
Trung Nguyen
Xem chi tiết
Linh Châu
Xem chi tiết
Incursion_03
Xem chi tiết
dbrby
Xem chi tiết
Thiều Khánh Vi
Xem chi tiết
Nguyễn Hiền
Xem chi tiết
Kuramajiva
Xem chi tiết
dbrby
Xem chi tiết
Thiều Khánh Vi
Xem chi tiết